Moments of Inertia, almost got it

In summary, the problem is to determine the moment of inertia of a structure consisting of three identical thin rods welded perpendicular to each other and rotated about an axis passing through one end of a rod and parallel to another. The known moments of inertia are (1/3ML^2) and (ML^2), and the third moment of inertia is yet to be found. The parallel-axis theorem may need to be applied, but further clarification is needed on the specific joining of the rods.
  • #1
sisigsarap
17
0
The problem is:

Three identical thin rods, each of length L and mass m, are welded perpendicular to one another. The assembly is rotated about an axis that passes through the end of one rod and is parallel to another. Determine the moment of intertia of this structure.

Ok what I know is that the rod which is connected to the axis of rotation has inertia of I = 1/3ML^2
And I know that the rod which is parallel to the axis has inertia of I = ML^2

So far I have (1/3ML^2) + (ML^2) and I just need one other moment of intertia which I am having difficulty finding.

I believe I need to apply the parallel-axis theorem, but I don't understand why.

If someone could please explain why you would use this theorem it would be very helpful! Thanks!
 
Physics news on Phys.org
  • #2
Can you elaborate? There are MANY ways to join three rods at right angles to each other. Are they joined at their midpoints, ends, something in between or other permutations?
 
  • #3
The three rods are joined at their midpoints.
 
  • #4
sisigsarap said:
The problem is:

Three identical thin rods, each of length L and mass m, are welded perpendicular to one another. The assembly is rotated about an axis that passes through the end of one rod and is parallel to another. Determine the moment of intertia of this structure.

Thanks!

do you mean the assembly looks some what like an xyz coordinate system with all of the centers of mass(midpoints) at the origin and rotating about one of the axes? If that's the case I don't think that the parallel axis theorem applies.
 
  • #5
Yes the assembly looks some what like an xyz coordinate system with all of the centers of mass (midpoints) at the center.
 

1. What is a moment of inertia?

A moment of inertia is a measure of an object's resistance to changes in its rotational motion. It is calculated by multiplying the mass of an object by the square of its distance from the axis of rotation.

2. How is moment of inertia different from mass?

Moment of inertia and mass are two different physical quantities. Mass is a measure of an object's resistance to linear motion, while moment of inertia is a measure of an object's resistance to rotational motion.

3. What factors affect the moment of inertia of an object?

The moment of inertia of an object is affected by its mass, shape, and distribution of mass relative to its axis of rotation. Objects with larger mass and more spread out mass distribution have higher moments of inertia.

4. How is moment of inertia useful in physics?

Moment of inertia is an important concept in physics, particularly in rotational motion. It helps to determine an object's angular acceleration and torque, and is used in many equations related to rotational motion.

5. How can I calculate the moment of inertia of an object?

The moment of inertia can be calculated using different formulas depending on the shape and distribution of the object's mass. For simple objects, such as a point mass or a solid cylinder, there are specific formulas. For more complex objects, the moment of inertia can be calculated by dividing the object into smaller parts and using the parallel axis theorem.

Similar threads

  • Introductory Physics Homework Help
Replies
2
Views
625
  • Introductory Physics Homework Help
Replies
3
Views
1K
  • Introductory Physics Homework Help
Replies
28
Views
541
Replies
25
Views
456
  • Introductory Physics Homework Help
Replies
11
Views
1K
  • Introductory Physics Homework Help
2
Replies
40
Views
2K
  • Introductory Physics Homework Help
Replies
21
Views
1K
  • Introductory Physics Homework Help
Replies
12
Views
954
  • Introductory Physics Homework Help
Replies
8
Views
1K
  • Introductory Physics Homework Help
Replies
4
Views
954
Back
Top